Question

lincar time invariant systcm (LTI), Determine whcther cach of the following systems is a wherc uis thc input and y is the out

0 0
Add a comment Improve this question Transcribed image text
Answer #1

Linean system , Aditvily Lineaniby Honogenity T Hemojehity Ta a T Liheanity Tanb +b T Spstent Ne The Sstem is neh-linean.GE NO shest aut System ts sald to be linean, the fellecoln conditions should Salfy a) No openations ae Perfomed on xC) inputshot cut TIV, the followirg System is Sald to be Conditions are sattstied. No openattohs ane Perfomed on bInitl Cohditiohs caFagc No Linean Time invaniantl (LTI) Esinu operation tme Component (NL- TV) Non-inean Time aVaniaht y(etsint time comonent CL

Add a comment
Know the answer?
Add Answer to:
lincar time invariant systcm (LTI), Determine whcther cach of the following systems is a wherc uis thc input and y is t...
Your Answer:

Post as a guest

Your Name:

What's your source?

Earn Coins

Coins can be redeemed for fabulous gifts.

Not the answer you're looking for? Ask your own homework help question. Our experts will answer your question WITHIN MINUTES for Free.
Similar Homework Help Questions
  • 7. Show that the following functions u(x, y) monic functions v(x, y) and determine f(z) = u(x,y) + iv(x, y) are harmoni...

    7. Show that the following functions u(x, y) monic functions v(x, y) and determine f(z) = u(x,y) + iv(x, y) are harmonic, find their conjugate har- as functions of 2. 2x2 2лу — 5х — 22. Зл? — 8ху — Зу? + 2у, (а) и(х, у) (b) и(х, у) (с) и(х, у) (d) u(a, y) 2e cos y 3e" sin y, = 3e-* cos y + 5e-" sin y, = elx cos y - e y sin y, (e) u(x,...

  • Linear time-invariant sys tems. Consider a SISO, LTI system: t(t) Az(t) + bu(t) y(t) cr (t)...

    Linear time-invariant sys tems. Consider a SISO, LTI system: t(t) Az(t) + bu(t) y(t) cr (t) with IC x(0) 3o. If the nominal input is a non-zero constant, i.e. if u(t)= u, under what conditions does there exist a constant nominal solu- tion (t) nominal output zero? Under what conditions do there exist constant nom- inal solutions that satisfy g = u for all u? To for some axo? Under what conditions is the corresponding

  • 3. (10 points) Two linear time invariant (LTI) systems with impulse response hi(k) and h2(k) are...

    3. (10 points) Two linear time invariant (LTI) systems with impulse response hi(k) and h2(k) are connected in cascade as shown in Figure 1. Let x(k) be the input, yı(k) be the output of the first LTI, and y2(k) be the output of the second LTI. Let hi(k) = k(0.7)k u(k), h2(k) = ku(k), and x(k) = (0.3)k u(k). Use z-transform to (a) find yı(k). (b) find y2(k). x(k) yi(k) y2(k) hi(k) h2(k)

  • 4. LTI Systems and Erponential Response. (12 pts) (a) (2 pts) Suppose an LTI system has...

    4. LTI Systems and Erponential Response. (12 pts) (a) (2 pts) Suppose an LTI system has input-output relationship y(t) 2r(t+3). What is the transfer function H(jw) of the given system. Show that H(jw)2. Hint: H(jw(tejdt (b) (5 pts) Suppose an LTI system has input-output relationship y(t)2r(t+3) as Problem 4-(a). Find the output y(t) using the complex exponential response method as discussed in lecture for the input r(t) = ej2t + 2 cos2(t). Hint: cos2(0) 1 (20 cos(26) an d 1-ejot...

  • LTI Systems. Consider two LTI subsystems that are connected in series

    (a) LTI Systems. Consider two LTI subsystems that are connected in series, where system Tl has step response s1(t)=u(t-1)-u(t-5) and system T2 has impulse response h2t = e-3tu(t). Find the overall impulse response h(t). Hint: you will need to find h1(t) first (b)Fourier Series. The input signal r(t) and impulse response h(t) of an LTI system are as follows:x(t) = sin(2t)cos(t)-ej3t +2 and h(t) = sin(2t)/t Use the Fourier Series method to find the output y(t) (c)Parseval's Identity and Theorem. Consider the system in the...

  • Q8) Consider the following causal linear time-invariant (LTI) discrete-time filter with input x[n...

    Q8) Consider the following causal linear time-invariant (LTI) discrete-time filter with input x[n] and output y[n] described by bx[n-21- ax[n-3 for n 2 0, where a and b are real-valued positive coefficients. A) Is this a finite impulse response (FIR) or infinite impulse response (IIR) filter? Why? B) What are the initial conditions and their values? Why? C) Draw the block diagram of the filter relating input x[n] and output y[n] D) Derive a formula for the transfer function in...

  • (c) If the impulse response function of a linear time invariant (LTI) system is h0)-Se u(),...

    (c) If the impulse response function of a linear time invariant (LTI) system is h0)-Se u(), compute the output of this system due to an input ) which is a 4 second pulse of height 3, as shown in Fig.1 below. x(t) t(sec) 0 Fig.1 Input signal 10 marks/

  • For a continuous time linear time-invariant system, the input-output relation is the following (x(t) the input, y(t) the...

    For a continuous time linear time-invariant system, the input-output relation is the following (x(t) the input, y(t) the output): , where h(t) is the impulse response function of the system. Please explain why a signal like e/“* is always an eigenvector of this linear map for any w. Also, if ¥(w),X(w),and H(w) are the Fourier transforms of y(t),x(t),and h(t), respectively. Please derive in detail the relation between Y(w),X(w),and H(w), which means to reproduce the proof of the basic convolution property...

  • Linear Time Invariant Systems 4] For each of the following continuous-time systems xt) is a real...

    Linear Time Invariant Systems 4] For each of the following continuous-time systems xt) is a real input. Determine whether the system is (1) stable, (2) causal, (3) linear and (4) time invariant (5% each): (a) T(x(t)] = sin(2π) x(t + 2%)-cos(2π) x(t-ro), where τ。> (b)T(x(t)] = x(4) (c) T(x(t)]Ξ14- AxzQ A is a complex constant.

  • 2.10. Window/modulator Consider the system where for an input x(t) the output is y(t) = x(oft)...

    2.10. Window/modulator Consider the system where for an input x(t) the output is y(t) = x(oft) for some function f(t). (a) Letf(t)=u(t)-11(t-10). Determine whether the system with input x(t) and output y(t)is linear, time invariant, and causal, Suppose x(t) = 4 cos(T/2), and f(t)=cos(67t/7) periodic? What frequencies are present in the output? Is this system linear? Is it time invariant? Explain. (b) and both are periodic. Is the output y(t) also (c) Let f(t) = u(t)-u (t-2) and the input...

ADVERTISEMENT
Free Homework Help App
Download From Google Play
Scan Your Homework
to Get Instant Free Answers
Need Online Homework Help?
Ask a Question
Get Answers For Free
Most questions answered within 3 hours.
ADVERTISEMENT
ADVERTISEMENT
ADVERTISEMENT